Inscription / Connexion Nouveau Sujet
Niveau LicenceMaths 2e/3e a
Partager :

convergence d'une série

Posté par
astroq123
07-07-19 à 20:50

Bonsoir,

s'il vous plait, pouvez-vous me proposer une méthode pour prouver la convergence de la série \sum_{n \geq 3} 3^{\left\lfloor{2\frac{\ln(n)}{\ln(\ln(n))}}\right\rfloor+1}/(\left\lfloor{2\frac{\ln(n)}{\ln(\ln(n))}}\right\rfloor+1)!

Merci

Posté par
Jezebeth
re : convergence d'une série 07-07-19 à 22:01

Bonsoir

Montrez que \dfrac{ln(n)}{ln(ln(n))}\rightarrow +\infty, par suite \left \lfloor \dfrac{ln(n)}{ln(ln(n))} \right \rfloor \sim \dfrac{ln(n)}{ln(ln(n))}.

Puis D'Alembert.

Posté par
Jezebeth
re : convergence d'une série 07-07-19 à 22:37

En notant a_n = \left \lfloor 2\dfrac{ln(n)}{ln(ln(n))} \right \rfloor +1, j'obtiens a_{n+1}-a_n= \dfrac{-2ln(nln(n))}{ln(ln(n+1))ln(ln(n))}+o(1), mais les factorielles seront gênantes pour D'Alembert, je ne sais pas si vous avez déjà essayé.
L'idée est généralement de tronçonner \mathbb{N} \cap [3,+\infty[ en intervalles simplifiant la partie entière, je ne sais pas si c'est faisable ici.

Posté par
astroq123
re : convergence d'une série 08-07-19 à 12:59

On peut remarquer que (Stirling) : \lambda^{\left \lfloor{\epsilon\frac{\ln(n)}{\ln(\ln(n))}}\right\rfloor+1}/(\left\lfloor{\epsilon\frac{\ln(n)}{\ln(\ln(n))}}\right\rfloor+1)! \sim (e\lambda)^{\left \lfloor{\epsilon\frac{\ln(n)}{\ln(\ln(n))}}\right \rfloor}/\sqrt{2 \pi \epsilon\frac{\ln(n)}{\ln(\ln(n))}}(\left\lfloor{\epsilon\frac{\ln(n)}{\ln(\ln(n))}}\right \rfloor+1)^{\left\lfloor{\epsilon\frac{\ln(n)}{\ln(\ln(n))}}\right\rfloor+1}

Posté par
Jezebeth
re : convergence d'une série 08-07-19 à 17:00

Bof, pour en faire quoi ?

Posté par
luzak
re : convergence d'une série 08-07-19 à 18:24

Bonsoir !
Ai-je bien lu qu'en posant a_n=\left\lfloor\dfrac2\log(n)}{\log(\log(n))}\right\rfloor tu veux étudier la série de terme général u_n=3^{b_n}\text{ avec } b_n=\dfrac{a_n+1}{(a_n+1)!}
Auquel cas tu as aussi u_n=\exp\Bigl(\dfrac{\log3}{(a_n)!}\Bigr) et l'équivalent de Stirling pourrait être utile.

Posté par
verdurin
re : convergence d'une série 08-07-19 à 18:49

Bonsoir,
si cette série est convergente, la convergence est très lente.

En reprenant l'idée de Jezebeth, à partir de n=7, an va prendre successivement toutes les valeurs entières supérieures à 5.

On a donc à étudier la convergence de \sum_{k\ge5}b_k \dfrac{3^k}{k!}b_k est le nombre de valeurs de n telles que a_n=k.

J'ai l'impression, mais je n'ai pas beaucoup cherché, que ce nombre n'est pas une fonction polynomiale de k.

Si on trouve une fonction polynomiale en k majorant b_k, c'est gagné.
Sinon il faut trouver d'autres pistes.

Posté par
astroq123
re : convergence d'une série 08-07-19 à 19:00

La série à étudier est \sum_{n \geq 3} \frac{3^{a_n}}{(a_n)!}a_n=\left\lfloor{2\frac{\ln(n)}{\ln(\ln(n))}}\right\rfloor+1

Posté par
Jezebeth
re : convergence d'une série 08-07-19 à 19:52

L'étude de x\mapsto x^{\frac{2}{ln(ln(x))}} inspire que b_k est majoré par e^e-e à partir d'un certain rang sans tenir compte de la faible croissance à l'infini vers l'infini, mais c'est loin d'être rigoureux et difficile de le formaliser.
Rien ne dit a priori que c'est polynomial lorsque k est grand pour cette partie droite.

Par contre pour D'Alembert, avec Sterling, peut-être fonctionne-ce (mais j'avoue avoir la flemme, ce doit être très laborieux !).

Posté par
Jezebeth
re : convergence d'une série 08-07-19 à 20:24

verdurin La question est donc : combien y a-t-il d'entiers dans I_k := [e ^{\frac{2k}{ln(k)}};e ^{\frac{2(k+1)}{ln(k+1)}}[ ?

La longueur d'I_k est :

e ^{\frac{2(k+1)}{ln(k+1)}}\left(1-e^{\frac{2k}{ln(k)}-\frac{2(k+1)}{ln(k+1)}} \right)=e ^{\frac{2(k+1)}{ln(k+1)}}\left(2\frac{ln(k)-1}{ln^2(k)}+\frac{2-ln(k)}{kln^3(k)}+O\left(\frac{1}{k^2} \right) \right) .

Cette piste-là non plus n'est donc pas à creuser, ça diverge violemment.

Posté par
Jezebeth
re : convergence d'une série 08-07-19 à 20:26

Oubliez le message précédent.

Posté par
luzak
re : convergence d'une série 09-07-19 à 08:14

J'ai dû lire à la "Shadoko" !
Inutile d'essayer la condition suffisante de d'Alembert puisque la condition suffisante de Cauchy donne une limite de \sqrt[n]{.} égale à 1.

Posté par
luzak
re : convergence d'une série 09-07-19 à 11:02

Il me semble qu'en utilisant l'équivalent de Stirling on obtient un équivalent en 1/n^2 :
Pour commencer, a_n\underset{n \to+\infty }{\quad\simeq\quad}\dfrac{2\log(n)}{\log(\log(n))}
La limite étant infinie, on a aussi \log(a_n)\underset{n \to+\infty }{\quad\simeq\quad}\log(\log n) puis a_n\,\log(a_n)\underset{n \to+\infty }{\quad\simeq\quad}\log(n^2)

u_n=\dfrac{3^{a_n}}{(a_n)!}\underset{n \to+\infty }{\quad\simeq\quad}\dfrac{\mathrm{e}^{a_n\log(3)}}{\sqrt{2\pi}\mathrm{e}^{(a_n+\frac12)\log(a_n)-a_n}}

L'exposant pour \sqrt{2\pi}\,u_n est donc
z_n=a_n(1+\log3)-\log(a_n)(a_n+\frac12)=-a_n\log(a_n)\Bigl(1+\dfrac1{2a_n}-\dfrac{1+\log3}{\log(a_n)}\Bigr)\underset{ n\to+\infty}{\quad=\quad}-\log(n^2)(1+o(1))}

et il en résulte \sqrt{2\pi}\,u_n\underset{ n\to+\infty}{\quad=\quad}\mathrm{e}^{-\log(n^2)}\mathrm{e}^{1+o(1)} donc u_n\underset{n \to+\infty }{\quad\simeq\quad}\dfrac{\mathrm{e}}{\sqrt{2\pi}\,n^2}

Posté par
astroq123
re : convergence d'une série 09-07-19 à 13:48

Pourquoi si z_n \sim -\log(n^2) alors e^{z_n} \sim \frac{1}{n^2}
(Il ne faut pas vérifier que \lim (z_n+\log(n^2))=0 pour avoir cela?)

Posté par
luzak
re : convergence d'une série 09-07-19 à 15:01

Tu as raison : j'aurais dû écrire z_n\underset{ n\to+\infty}{\quad=\quad}-\log(n^2)(1+o(1))} puis \mathrm{e}^{z_n}\underset{ n\to+\infty}{\quad=\quad}\mathrm{e}^{-\log(n^2)(1+o(1))} ce qui ne fait pas le résultat écrit (j'ai fait la bêtise \mathrm{e}^{ab}=\mathrm{e}^a\,\mathrm{e}^b !!!)

Bref il faudrait pour conclure un terme supplémentaire dans le développement asymptotique de a_n.
Désolé, je m'y remets !

Posté par
Jezebeth
re : convergence d'une série 21-07-19 à 17:21

astroq123, tu as pu avancer sur cet exercice ?

Posté par
astroq123
re : convergence d'une série 02-02-20 à 12:58

On peut chercher la limite n^{\frac{3}{2}} \frac{3^{u_n}}{(u_n)!}=exp(u_n(1+\ln(3))-\ln(u_n)(u_n+\frac{1}{2})+\frac{3}{2}\ln(n))

On a u_n(1+\ln(3))-\ln(u_n)(u_n+\frac{1}{2})+\frac{3}{2}\ln(n)=\ln(n)(u_n(1+\ln(3))/\ln(n)-\ln(u_n)(u_n+\frac{1}{2})/\ln(n)+\frac{3}{2})
Avec \lim_n \ u_n(1+\ln(3))/\ln(n)=0, \ \lim_n \ \ln(u_n)(u_n+\frac{1}{2})/\ln(n)=2

D'ou la limite est \lim_n n^{\frac{3}{2}} \frac{3^{u_n}}{(u_n)!}=0

et la serie converge

Posté par
astroq123
re : convergence d'une série 02-02-20 à 13:23

On pose u_n :=\left \lfloor{2ln(n)/ln(ln(n))}\right \rfloor



Vous devez être membre accéder à ce service...

Pas encore inscrit ?

1 compte par personne, multi-compte interdit !

Ou identifiez-vous :


Rester sur la page

Inscription gratuite

Fiches en rapport

parmi 1674 fiches de maths

Désolé, votre version d'Internet Explorer est plus que périmée ! Merci de le mettre à jour ou de télécharger Firefox ou Google Chrome pour utiliser le site. Votre ordinateur vous remerciera !